Discussion

Which one of the following is an assumption on which the argument depends?
(A)All neurochemical changes produce corresponding psychological changes.
(B)...
(C)...
(D)...
(E)...
(F)...
*This question is included in June 2012 LSAT (PT66): Logical Reasoning A, question #4

The solution is

Posted: 01/10/2013 21:01
I dont understand why A is incorrect and why B is correct. Doesntvthe passage need the assumption that "talk" therapy actually treats the behaviorbyou want treated?

You need to be signed in to perform that action.

Sign In